LSAT and Law School Admissions Forum

Get expert LSAT preparation and law school admissions advice from PowerScore Test Preparation.

User avatar
 Dave Killoran
PowerScore Staff
  • PowerScore Staff
  • Posts: 5853
  • Joined: Mar 25, 2011
|
#46272
Complete Question Explanation
(The complete setup for this game can be found here: lsat/viewtopic.php?t=14920)

The correct answer choice is (A)

During the analysis of the final rule, we concluded that if J is on team 1, then R and F must be on team 2:
O97_Game_#3_#13_diagram 1.png

Hence, answer choice (A) is correct.
You do not have the required permissions to view the files attached to this post.
User avatar
 Kenyambo
  • Posts: 8
  • Joined: May 17, 2023
|
#101912
I still do not understand how you got to Choice A. Choices B-D are easy in my mind to eliminate simply because it does not contain Rice (Rule #4) but I am still stuck in between Choice A and Choice E .. PLEASE HELP!
 Luke Haqq
PowerScore Staff
  • PowerScore Staff
  • Posts: 747
  • Joined: Apr 26, 2012
|
#101939
Hi Kenyambo!

To see why (A) must be true, to start, we're told that J is on team 1. This tells us that R must be on team 2.

1: J, _, _
2: R, _, _
The R makes team 2 somewhat restricted. Since R is on the team, it can't include N (rule 2), and it also can't include M (rule 3). R is a linguist, and we're told that a team must contain at least 1 linguist and at least 1 anthropologist. That leaves F and J as possible as the possible anthropologist (since M was eliminated). However, J is already on team 1. This means that F must be on team 2 with R, reflected in answer choice (A).

Get the most out of your LSAT Prep Plus subscription.

Analyze and track your performance with our Testing and Analytics Package.